Re: Controesempi in Analisi

Messaggioda quantunquemente » 21/06/2015, 13:08

un controesempio sul teorema di Rolle

$y=|x|$ in $[-1,1]$
la funzione è continua nell'intervallo,si ha $f(-1)=f(1)$,ma è derivabile solo in $(-1,1)-{0}$

non esiste un punto interno all'intervallo in cui si annulli la derivata
quantunquemente
Cannot live without
Cannot live without
 
Messaggio: 376 di 3204
Iscritto il: 14/04/2015, 15:24

Re: Controesempi in Analisi

Messaggioda Martino » 01/07/2015, 23:53

Esempio di funzione derivabile ma non \( \displaystyle \mathcal{C}^1 \) (cioè la derivata non è continua).
\[
f(x) = \begin{cases}
x^2\sin(1/x), & se\ x\neq 0,\\
0, & se\ x = 0.
\end{cases}
\]
La derivata di questa funzione non è continua in zero.

Esempio di funzione con derivata positiva in un punto ma non crescente in nessun intorno di tale punto.
\[
f(x) = \begin{cases}
x^2\sin(1/x)+x, & se\ x\neq 0,\\
0, & se\ x = 0.
\end{cases}
\]
La derivata in zero vale 1 ma la funzione non è crescente in nessun intorno di zero.
Le persone che le persone che le persone amano amano amano.
Avatar utente
Martino
Moderatore globale
Moderatore globale
 
Messaggio: 6173 di 13035
Iscritto il: 21/07/2007, 10:48
Località: Brasilia

Re: Controesempi in Analisi

Messaggioda mazzarri » 17/07/2015, 18:43

quantunquemente ha scritto:un controesempio sul teorema di Rolle

$y=|x|$ in $[-1,1]$
la funzione è continua nell'intervallo,si ha $f(-1)=f(1)$,ma è derivabile solo in $(-1,1)-{0}$

non esiste un punto interno all'intervallo in cui si annulli la derivata


ciao caro non capisco il tuo contro-esempio... il teorema di Rolle dice che:

se una funzione è continua e derivabile in un intervallo $(a,b)$ e se $f(a)=f(b)$ allora esiste almeno un punto $c in (a,b)$ tale che $f'(c)=0$

ma la tua funzione $|x|$ non è derivabile in $RR$ come tu stesso dici... in $x=0$ ha un punto angoloso per cui cadono le ipotesi del teorema di Rolle che quindi non è applicabile

ciao!
Ultima modifica di mazzarri il 17/07/2015, 19:05, modificato 1 volta in totale.
mazzarri
Advanced Member
Advanced Member
 
Messaggio: 743 di 2904
Iscritto il: 08/10/2014, 20:49
Località: Torino

Re: Controesempi in Analisi

Messaggioda quantunquemente » 17/07/2015, 18:58

ciao
il mio controesempio segue l'indicazione scritta da gugo nel punto 1) del suo primo post
quantunquemente
Cannot live without
Cannot live without
 
Messaggio: 614 di 3204
Iscritto il: 14/04/2015, 15:24

Re: Controesempi in Analisi

Messaggioda mazzarri » 17/07/2015, 19:06

[/quote="quantunquemente"]ciao
il mio controesempio segue l'indicazione scritta da gugo nel punto 1) del suo primo post[/quote]


:smt023 :smt023
mazzarri
Advanced Member
Advanced Member
 
Messaggio: 744 di 2904
Iscritto il: 08/10/2014, 20:49
Località: Torino

Re: [CiA] Il serpente topologico

Messaggioda Platone » 03/01/2017, 14:07

Salve a tutti. Riscrivo con molto piace su questo forum dopo credo 10 anni! :D
Non so se l'utente j18eos frequenta ancora il forum. Stavo leggendo questo topic è qualcosa non mi torna nella dimostrazione riportata sotto. Ovviamente la domanda è posta a tutti gli utenti :wink:

j18eos ha scritto: Il serpente topologico è un insieme connesso ma non connesso per cammini!

Dimostrazione. Sia \(\displaystyle(0;y_1)\in\overline{X}\); per assurdo esista una funzione continua \(\displaystyle\gamma\) da \(\displaystyle[0;1]\) a \(\displaystyle\overline{X}\) tale che \(\displaystyle\gamma(0)\in X\) e \(\displaystyle\gamma(1)=(0;y_1)\).
Posto:
\[
p_1:(x;y)\in\overline{X}\to x\in[0;+\infty[
\]
si consideri:
\[
\sup\{t\in[0;1]\mid p_1(\gamma(t))>0\}=\sup X_+=\tau
\]
per ipotesi \(\displaystyle0\leq\tau<1\) ovvero essendo \(\tau\) il minimo dei maggioranti di \(\displaystyle X_+\) si ha che:
\[
\forall t\in]\tau;1],\,p_1(\gamma(t))=0.
\]
Ricordata la definizione dell'estremo superiore per l'insieme \(\displaystyle X_+\):
\[
\forall t\in X_+,\,t\leq\tau,\\
\forall\epsilon>0,\,\exists t\in X_+\mid\tau-\epsilon\leq t\leq\tau
\]
per il teorema della permanenza del segno delle funzioni continue:
\[
\exists\delta>0:\forall t\in]\tau-\delta;\tau+\delta[\cap[0;1],\,p_1(\gamma(t))>0
\]
quindi per la seconda proprietà che definisce l'estremo superiore di un insieme, dev'essere \(\tau=1\) in assurdo con quanto affermato!

Onde evitare l'assurdo \(\displaystyle\overline{X}\) non è un insieme connesso per cammini.


Non mi torna la parte finale del ragionamento, ossia il modo in cui vine applicato il teorema della permanenza del segno delle funzioni continue. D'accordo che \(p_1 \circ \gamma \) è una funzione continua, ma per poter applica il teorema dovrebbe essere \( p_1(\gamma(\tau))>0 \), ma non mi pare sia così in questo caso; infatti \(\tau\) non appartiene all'insieme \(X_+\) (altrimenti sarebbe il massimo di quell'insieme, mentre il supp in generale è solo il minimo dei maggioranti, e come tale può non appartenere all'insieme).
Sbaglio in qualcosa?
Accetto volentieri pareri :)
Non ho mai conosciuto un matematico che sapesse ragionare. (Platone)
Avatar utente
Platone
Average Member
Average Member
 
Messaggio: 465 di 572
Iscritto il: 16/08/2005, 10:47

Re: Controesempi in Analisi

Messaggioda Leonardo97 » 12/12/2019, 22:30

Un noto teorema di analisi 1 asserisce che se $f: I \subseteq \mathbb{R} \to \mathbb{R}$ è una funzione definita sull'intervallo $I$, continua e invertibile, allora anche la sua inversa $f^{-1}$ è continua nel rispettivo dominio.
Tale teorema non è più vero se $I$ non è un intervallo.
Come controesempio consideriamo la funzione:
\[f: D=[0,1) \cup [2,3] \to \mathbb{R} \quad | \quad f(x)=
\begin{cases}
x \quad \text{se} \quad x \in [0,1)\\
x-1 \quad \text{se} \quad x \in [2,3]\\
\end{cases}
\]
Ovviamente $f$ è continua e invertibile nel suo dominio $D$, con inversa:
\[f^{-1}: [0,1) \cup [1,2] \to [0,1) \cup [2,3] \quad | \quad f^{-1}(y)=
\begin{cases}
y \quad \text{se} \quad y \in [0,1)\\
y+1 \quad \text{se} \quad y \in [1,2]\\
\end{cases}
\]
Tuttavia $f^{-1}$ non è continua nel punto $x=1$ dato che:
\[\lim_{y \to 1^-} f^{-1}(y)=\lim_{y \to 1^-} y=1\]
mentre:
\[\lim_{y \to 1^+} f^{-1}(y)=\lim_{y \to 1^+} (y+1)=2\]
e quindi limite destro e sinistro non coincidono (discontinuità a salto nel punto $x=1$).

Quanto detto può sembrare banale, ma talvolta durante un esame può capitare di trascurare l'ipotesi che $I$ sia un intervallo.
Per $n$ che va a infinito siamo tutti morti.
Avatar utente
Leonardo97
Junior Member
Junior Member
 
Messaggio: 25 di 218
Iscritto il: 20/09/2019, 17:33
Località: Viterbo

Re: Controesempi in Analisi

Messaggioda Leonardo97 » 13/12/2019, 14:46

Una funzione reale di variabile reale $f: I=(x_0-r,x_0+r) \subseteq \mathbb{R} \to \mathbb{R}$ si dice analitica in $I$ (e si scrive $f \in C^{\omega}(I)$) se è infinitamente derivabile in $I$ (cioè $f \in C^{\infty}(I)$) e se coincide in $I$ con la sua serie di Taylor$:
\[f(x)=\sum_{n=0}^{\infty} \frac{f^{(n)}(x_0)}{n!}(x-x_0)^n \quad \forall x \in I\]
Tuttavia, il fatto che $f$ sia infinitamente derivabile non implica necessariamente l'analiticità, cioè la sua serie di Taylor potrebbe non convergere, o convergere a un valore diverso da $f(x)$ (insomma $C^{\omega} \subset C^{\infty}$ strettamente).
Per mostrare ciò, si ricorre spesso al seguente facile controesempio:
\[f: \mathbb{R} \to \mathbb{R} \quad | \quad f(x)=
\begin{cases}
e^{-\frac{1}{x^2}} \quad \text{se} \quad x \neq 0\\
0 \quad \text{se} \quad x=0\\
\end{cases}
\]
Mostriamo che $f$ è infinitamente derivabile nel punto $x_0=0$ con derivate tutte nulle.

Chiaramente la funzione è infinitamente derivabile in $\mathbb{R} \setminus \{0\}$. Fissiamo un punto generico $x \in \mathbb{R} \setminus \{0\}$.
Dimostriamo per induzione che $f^{(n)}(x)=P_n(\frac{1}{x})e^{-\frac{1}{x^2}} \quad \forall n \in \mathbb{N}$ dove $P_n(\frac{1}{x})$ è un polinomio di grado $3n$.
Ovviamente:
\[n=1 \Longrightarrow f'(x)=\frac{2}{x^3}e^{-\frac{1}{x^2}}\]
Inoltre supponendo vero che $f^{(n)}(x)=P_n(\frac{1}{x})e^{-\frac{1}{x^2}}$ (ipotesi induttiva) risulta:
\[f^{(n+1)}(x)=\frac{d}{dx}\left[P_n\left(\frac{1}{x}\right)\right]e^{-\frac{1}{x^2}}+\frac{2}{x^3}P_n\left(\frac{1}{x}\right)e^{-\frac{1}{x^2}}=\\
\\
=\frac{d}{dx}\left[a_{3n}\left(\frac{1}{x}\right)^{3n}+\dots+a_1\frac{1}{x}+a_0\right]e^{-\frac{1}{x^2}}+\\
\\
+\left[2a_{3n}\left(\frac{1}{x}\right)^{3n+3}+\dots+2a_1\left(\frac{1}{x}\right)^4+2a_0\left(\frac{1}{x}\right)^3\right]e^{-\frac{1}{x^2}}=\\
\\
=\left[-3na_{3n}\left(\frac{1}{x}\right)^{3n-1+2}-\dots-a_1\left(\frac{1}{x}\right)^2\right]e^{-\frac{1}{x^2}}+\\
\\
+\left[2a_{3n}\left(\frac{1}{x}\right)^{3n+3}+\dots+2a_1\left(\frac{1}{x}\right)^4+2a_0\left(\frac{1}{x}\right)^3\right]e^{-\frac{1}{x^2}}=\\
\\
=\left[2a_{3n}\left(\frac{1}{x}\right)^{3(n+1)}+\dots-a_1\left(\frac{1}{x}\right)^2\right]e^{-\frac{1}{x^2}}=P_{n+1}\left(\frac{1}{x}\right)e^{-\frac{1}{x^2}}\]
con $P_{n+1}\left(\frac{1}{x}\right)$ polinomio di grado $3(n+1)$.
Dunque adesso risulta:
\[\lim_{x \to 0} f^{(n)}(x)=\lim_{x \to 0} P_n\left(\frac{1}{x}\right)e^{-\frac{1}{x^2}}=\lim_{x \to 0} \left[a_{3n}\left(\frac{1}{x}\right)^{3n}+\dots+a_1\left(\frac{1}{x}\right)+a_0\right]e^{-\frac{1}{x^2}}=\\
\\
=\lim_{x \to 0} \frac{1}{x^{3n}}\left(a_{3n}+\dots+a_1x^{3n-1}+a_0x^{3n}\right)e^{-\frac{1}{x^2}}=\\
\\
=a_{3n}\lim_{x \to 0} \frac{\frac{1}{x^{3n}}}{e^{\frac{1}{x^2}}}=a_{3n}\lim_{t \to +\infty} \frac{t^{\frac{3n}{2}}}{e^t}=0 \quad \forall n \in \mathbb{N}\]
avendo posto $t=\frac{1}{x^2}$.
Quindi per un noto teorema dell'analisi $\exists f^{(n)}(0)=\lim_{x \to 0} f^{(n)}(x)=0 \quad \forall n \in \mathbb{N}$.
Quindi abbiamo che la serie di Taylor di $f$ nel punto $x_0=0$ è data da:
\[\sum_{n=0}^{\infty} \frac{f^{(n)}(0)}{n!}x^n=0 \quad \forall x \in \mathbb{R}\]
Quindi $f$, pur essendo infinitamente derivabile in $\mathbb{R}$, non coincide con la sua serie di Taylor (se non nel punto $x_0=0$ dove $f(x_0)=0$) e non è dunque analitica.
Per $n$ che va a infinito siamo tutti morti.
Avatar utente
Leonardo97
Junior Member
Junior Member
 
Messaggio: 26 di 218
Iscritto il: 20/09/2019, 17:33
Località: Viterbo

Re: Controesempi in Analisi

Messaggioda 3m0o » 12/01/2021, 01:40

Contribuisco ai controesempi in Analisi con i seguenti:
Lipschitz ma non uniformemente continua
È un risultato noto che se \( I \subseteq \mathbb{R} \) è un intervallo ed \( f: I \to \mathbb{R} \) è \(k\)-lipschitz allora è uniformemente continua.

Dimostrazione:
Testo nascosto, fai click qui per vederlo
Sia \( \epsilon >0 \), e prendiamo \( \delta = \frac{\epsilon}{k} \) risulta che per ogni \(x,y \in I \) tale che \( \left| x- y \right| \leq \delta \) allora abbiamo
\[ \left| f(x) - f(y) \right| \leq k \delta = \epsilon \]

Potremmo essere tentati di pensare che quanto segue sia vero.
Sia \( f: \mathbb{R} \to \mathbb{R}\) tale che per ogni \( [a,b] \subset \mathbb{R} \) risulta che \( f : [a,b] \to \mathbb{R} \) è \(k\)-lipschitziana allora \(f \) è uniformemente continua su \( \mathbb{R} \).
Invero risulta falso.

Contro-esempio
Testo nascosto, fai click qui per vederlo
\( f(x) = x^2 \) è lipschitziana con costante \(k= 2 \max( \left| a \right|, \left| b \right| ) \) su tutti gli intervalli \( [a,b] \subset \mathbb{R} \), ma non è uniformemente continua. Infatti per \( \epsilon = 1 \) abbiamo che per ogni \( \delta >0 \) esistono \( x, y \) tale che \( \left| x-y \right| \leq \delta \) ma
\[ \left| x^2-y^2 \right| \geq 2 \geq 1 \]
Infatti
\[ \left| x^2-y^2 \right| = \left| x- y \right| \left| x+y \right| \]
e prendendo \( x= \frac{2}{\delta} \) e \( y= \frac{2+\delta^2}{\delta} \) abbiamo che
\[ \delta \left| \frac{4+\delta^2}{\delta} \right| = \left| 4 + \delta^2 \right| \geq 2 > 1 \]




Teorema di Dini
Sia \( I=[a,b] \), e sia \( (f_n)_{n \geq 0} \) una successioni di funzioni \( f_n : I \to \mathbb{R} \) continue e crescenti, i.e. \( f_{n+1}(x) \geq f_n(x) \) per ogni \( n \geq 0 \). Tale che
\[ f_n \xrightarrow{n \to \infty} f \ \ \ \ \ \text{ puntualmente} \]
allora se \(f \) è continua abbiamo
\[ f_n \xrightarrow{n \to \infty} f \ \ \ \ \ \text{ uniformemente} \]

Togliamo l'ipotesi \( (f_n)_{n \geq 0} \) è una successione crescente.
contro-esempio:
Testo nascosto, fai click qui per vederlo
Sia allora \( f_n : [0,1] \to \mathbb{R} \) definita da
\[ f_n(x) = \left\{\begin{matrix}
nx& \text{se} & x \in [0,1/n] \\
1-n\left( x- \frac{1}{n} \right)& \text{se} & x \in [1/n,2/n] \\
0 & \text{altrimenti}&
\end{matrix}\right. \]
Allora abbiamo che \( \sup_{x \in [0,1] } \left| f_n(x) - 0 \right| = 1 \) ma \( f_n \to 0 \) puntualmente.


Togliamo l'ipotesi \( I=[a,b] \) e prendiamo \( I=(0,1]\).
contro-esempio:
Testo nascosto, fai click qui per vederlo
Sia allora \( f_n : (0,1] \to \mathbb{R} \) definita da
\[ f_n(x) = \frac{-1}{(n+1)x} \to 0 \ \ \ \ \ \text{ puntualmente} \]
Ma
\[ \sup_{x \in I} \left| \frac{-1}{(n+1)x} - 0 \right| = \infty \]


Togliamo l'ipotesi \( f \) continua
contro-esempio:
Testo nascosto, fai click qui per vederlo
Sia allora \( f_n : [0,1] \to \mathbb{R} \) definita da \( f_n(x) = - x^n \). Abbiamo che
\[ f_n \to f \]
definita da
\[ f(x) = \left\{\begin{matrix}
0& \text{se} & x < 1 \\
-1& \text{se}& x=1
\end{matrix}\right. \]
Ma
\[ \sup_{ x \in [0,1] } \left| f_n(x) - f(x) \right| = 1 \]


Permutazioni illegali di un integrale con il limite
Sia \( (f_n)_{n \in \mathbb{N}} \) delle funzioni continue su \( [a,b] \) tale che \( (f_n)_{n \geq 0} \) converge uniformemente verso una funzione \( f:[a,b] \to \mathbb{R} \) allora \(f \) è continua e inoltre
\[ \int_a^b f_n(x) dx \xrightarrow{n \to \infty} \int_a^b f(x) dx \]

Potremmo essere tentati di credere che il teorema sia valido se sostituiamo convergenza uniforme con convergenza puntuale
Contro-esempio:
Testo nascosto, fai click qui per vederlo
Sia allora \( f_n : [0,1] \to \mathbb{R} \) definita da
\[ f_n(x) = \left\{\begin{matrix}
n^2x& \text{se} & x \in [0,1/n] \\
n-n^2\left( x- \frac{1}{n} \right)& \text{se} & x \in [1/n,2/n] \\
0 & \text{altrimenti}&
\end{matrix}\right. \]

Abbiamo che \( f_n \to 0 \) puntualmente ma
\[ \int_0^1 f_n(x) dx = 1 \neq 0 = \int_0^1 0 dx \]


Potremmo anche essere tentati di pensare che se \( f_n \to f \) puntualmente ma non uniformemente allora
\[ \int_a^b f_n (x) dx \nrightarrow \int_a^b f(x) dx \]
Controesempio:
Testo nascosto, fai click qui per vederlo
Sia \( f_n : [0,1] \to \mathbb{R} \) definita da \( f_n(x) = x^n \) e sia \( f: [0,1] \to \mathbb{R} \) definita da
\[ f(x) = \left\{\begin{matrix}
0& \text{se} & x < 1 \\
1& \text{se}& x=1
\end{matrix}\right. \]
abbiamo che \( f_n \to f \) puntualmente ma non uniformemente. Nonostante ciò
\[ \int_0^1 x^n dx = \left.\begin{matrix}
\frac{x^{n+1}}{n+1}
\end{matrix}\right|_0^1 = \frac{1}{n+1} \xrightarrow{n \to \infty} 0 \]
e
\[ \int_0^1 f(x) dx = 0 \]



Limiti e limiti per successioni
Sia \(f : U \to \mathbb{R} \) definita in un intorno bucato di \(x^{\ast} \) allora
\[ \lim_{x \to x^{\ast}} f(x) = \ell \]
se e solo se per ogni \( (x_n)_{n \geq 0 } \) tale che \(U \ni x_n \neq x^{\ast} \) per ogni \( n \geq 0 \) e tale che \( x_n \to x^{\ast} \) risulta che
\[ \lim_{n \to \infty} f(x_n) = \ell \]


Togliamo l'ipotesi \( x_n \neq x^{\ast} \).
contro-esempio:
Testo nascosto, fai click qui per vederlo
\[ f(x) = \left\{\begin{matrix}
1 & \text{se} & x = 0 \\
x& \text{se}& x \neq 0
\end{matrix}\right. \]
allora \(x_n = 0 \) per ogni \(n \geq 0 \) è una successione tale che \( x_n \to 0 \) ma
\[ \lim_{n \to \infty} f(x_n) = 1 \neq 0 = \lim_{ x \to 0} f(x) \]


Le somme infinite non sono commutative
Se \( a_k \geq 0 \) per ogni \( k \geq 0 \), allora per ogni permutazione \( \sigma : \mathbb{N} \to \mathbb{N} \) abbiamo
\[ \sum_{k=0}^{\infty} a_{\sigma(k)} = \sum_{k=0}^{\infty} a_k \]
In particolare se \( (b_k)_{k \geq 0 } \) è una successione qualsiasi e
\[ \sum_{k=0}^{\infty} b_k \]
è assolutamente convergente allora
\[ \sum_{k=0}^{\infty} b_k = \sum_{k=0}^{\infty} b_{\sigma(k)} \]
per ogni permutazione \( \sigma : \mathbb{N} \to \mathbb{N} \).

Potremmo essere tentati di togliere sostituire la convergenza assoluta di
\[ \sum_{k=0}^{\infty} b_k \]
con la convergenza (semplice).
Controesempio:
Testo nascosto, fai click qui per vederlo
\[ \sum_{k=1}^{\infty} \frac{(-1)^k}{k} = - \ln(2) \]
possiamo trovare una permutazione \( \sigma \) tale che
\[ \sum_{k=1}^{\infty} \frac{(-1)^{\sigma(k)}}{\sigma(k)} = \infty \]
Definiamo la permutazione
\( \sigma(1) = 1, \sigma(2) = 2 , \sigma(3)=4, \sigma(4)=3, \sigma(5)=6, \sigma(6)=8 , \ldots \) in modo tale da guadagnare 1/4 ogni volta e non perdere nulla. Riarrangiamo i termini dunque
\[ -1 + \frac{1}{2} + \frac{1}{4} - \frac{1}{3} + \frac{1}{6} + \frac{1}{8} + \frac{1}{10} + \frac{1}{12} - \frac{1}{5} + \frac{1}{14} + \ldots + \frac{1}{28} - \frac{1}{7} + \ldots\]
\[ \geq -1 + \frac{1}{4} + \frac{1}{4} + \frac{1}{4} + \ldots = - 1 + \sum_{k=1}^{\infty} \frac{1}{4} = \infty \]

In effetti si può fare di più, se \( a_k \to 0 \) con
\[ \sum_{k=0}^{\infty} a_k^+ = \infty \]
e
\[ \sum_{k=0}^{\infty} a_k^{-} = - \infty \]
dove \( a_k^+ \) e rispettivamente \( a_k^- \) sono i termini positivi e rispettivamente negativi della successione \( a_k \). Allora per ogni \( r \in \mathbb{R} \), esiste una permutazione \( \sigma_r : \mathbb{R} \to \mathbb{R} \) tale che
\[ \sum_{k=0}^{\infty} a_{\sigma_r(k)} = r \]

Al lettore lasciamo l'esercizio di costruire una permutazione \( \sigma_{\pi} \) tale che
\[ \sum_{k=1}^{\infty} \frac{(-1)^{\sigma_{\pi}(k)}}{\sigma_{\pi}(k)} = \pi \]


Criterio di Leibniz
Sia \( (a_k)_{k \geq 0} \) una successione tale che
\[ a_k \xrightarrow{ k \to \infty} 0 \]
\[ a_k a_{k+1} \leq 0 \ \ \ \ \forall k \]
\[ \left| a_{k+1} \right| \leq \left| a_k \right| \]
allora
\[ \sum_{k=0}^{\infty} a_k \]
converge

Togliamo l'ipotesi che \( \left| a_{k+1} \right| \leq \left| a_k \right| \).
Contro-esempio:
Testo nascosto, fai click qui per vederlo
\[ a_k := \frac{ \left( (-1)^k + 1/2 \right)}{k+1} \]
abbiamo che soddisfa tutte le altre condizioni ma
\[ \sum_{k = 0 }^{N } a_k \xrightarrow{N \to \infty} \infty \]
3m0o
Cannot live without
Cannot live without
 
Messaggio: 1710 di 5323
Iscritto il: 02/01/2018, 15:00

Precedente

Torna a Analisi matematica di base

Chi c’è in linea

Visitano il forum: Nessuno e 1 ospite